Course Content
All Previous Years Krok 2 Papers with Explanations
About Lesson

Question From (1 To 50 )

1. A 48-year-old man was brought to the doctor in an unconscious state. A history of alcohol abuse. During the examination, the skin is pale, there are “vascular stars” on the body, a liver odor from the mouth, and the tongue is covered with a brown coating. The subcutaneous veins of the front abdominal wall are dilated in the form (caput medusae). When palpating the organs of the abdominal cavity, there is a positive symptom of fluctuation, hepatomegaly, splenomegaly. Swelling of the lower extremities. What is the most probable condition of the patient?

A. Hypoglycemic coma

B. Acute gastrointestinal bleeding

C. Acute liver failure

D. Hyperglycemic coma

E. Perforating stomach ulcer


Answer:  Acute liver failure

Explanation

The most probable condition of the patient is acute liver failure, which is suggested by several clinical findings. The patient has a history of alcohol abuse, which is a common cause of liver damage leading to liver failure.   The presence of “vascular stars” on the body, a liver odor from the mouth, a brown coating on the tongue, and dilated subcutaneous veins in the form of caput medusae are all indicative of liver cirrhosis, which can ultimately lead to acute liver failure.  

Furthermore, the positive symptom of fluctuation, hepatomegaly, and splenomegaly suggest the presence of ascites, which is a common complication of liver cirrhosis. The swelling of the lower extremities is also a common finding in patients with liver failure due to increased pressure in the veins of the legs.  

Hypoglycemic and hyperglycemic comas are unlikely to be the cause of the patient’s symptoms, as they do not typically present with the clinical findings described. Acute gastrointestinal bleeding and perforating stomach ulcers can also cause abdominal pain and other symptoms, but they are less likely to present with the combination of findings seen in this patient.


2. A 58-year-old woman with sensorineural deafness as a result of a complication after purulent meningitis plans to install a cochlear implant. Vaccination against which infection is necessary to prevent infectious diseases of the central nervous system and hearing organs? Select one:

A. Tick-borne encephalitis virus

B. Tuberculosis after a negative Mantoux test result

C. Pneumococcal infection

D. Poliomyelitis

E. Hemophilic infection


Answer: Pneumococcal infection

Explanation

The vaccination that is necessary to prevent infectious diseases of the central nervous system and hearing organs before installing a cochlear implant is pneumococcal vaccination.   Purulent meningitis is a serious bacterial infection that can cause inflammation of the meninges, the membranes that surround the brain and spinal cord. It can lead to severe complications, including sensorineural deafness. Cochlear implantation is a common treatment for individuals with sensorineural deafness.  

Pneumococcal infection is a common cause of bacterial meningitis and can also cause other serious infections, such as pneumonia, sepsis, and otitis media, which can affect the hearing organs. Therefore, vaccination against pneumococcal infection is necessary to prevent infectious diseases of the central nervous system and hearing organs.  

Tick-borne encephalitis virus, tuberculosis, poliomyelitis, and Hemophilic infection are not directly related to the central nervous system or hearing organs, and therefore, vaccination against these infections is not necessary for preventing infectious diseases of the central nervous system and hearing organs before installing a cochlear implant.  of choice to directly assess the condition of the coronary arteries and identify any obstructive lesions.


3. A 47-year-old man is a worker in a weaving workshop, has 15 years of experience in this industry, and works in conditions of highfrequency, intense noise. During a periodic medical examination, he was diagnosed with “occupational deafness”. What is the basis for such a diagnosis?

A. Work experience in this production

B. Results of the study of the condition of the inner ear

C. Data of audiometry and hygienic assessment of working conditions

D. Characteristics of noise at this production

E. The results of the study of indicators of the central nervous system


Answer: Data from audiometry and hygienic assessment of working conditions

Explanation

The basis for the diagnosis of “occupational deafness” in the worker in the weaving workshop is the data from audiometry and hygienic assessment of working conditions.   Occupational deafness is a type of sensorineural hearing loss caused by exposure to loud noise in the workplace over an extended period. In this particular case, the worker has 15 years of experience in the weaving industry, which is a high-frequency, intense noise environment that can cause hearing damage.  

Audiometry is a diagnostic test that measures the ability to hear sounds and is used to assess the extent of hearing loss. In this case, the results of audiometry would be used to confirm the diagnosis of occupational deafness.   Hygienic assessment of working conditions is another important factor in the diagnosis of occupational deafness.

It involves measuring the level of noise exposure in the workplace and comparing it to established safety standards. If the noise levels in the weaving workshop are found to be above the recommended levels, it can contribute to hearing damage and lead to a diagnosis of occupational deafness.   Therefore, the correct answer is C, data from audiometry and hygienic assessment of working conditions.  


4. hygienic assessment of working conditions After a Q-myocardial infarction, the patient is troubled by shortness of breath with minor exertion, night attacks of dry cough, a feeling of wheezing in the chest and orthopnea. During the examination: acrocyanosis, heart rate – 96/min., tachycardia, weakening of the 1st tone above the apex, the 3rd tone is heard. Voiceless wheezing in the lungs in the basal parts. During echocardioscopy, dilatation of the left ventricle, thinning and dyskinesia of the interventricular septum, ejection fraction – 39%. What variant of myocardial dysfunction is the most likely? Select one:

A. Mixed dysfunction of both ventricles

B. Diastolic dysfunction of the right ventricle

C. Systolic dysfunction of the right ventricle

D. Diastolic dysfunction of the left ventricle

E. Systolic dysfunction of the left ventricle


Answer: Systolic dysfunction of the left ventricle

Explanation

The most likely variant of myocardial dysfunction in this patient is systolic dysfunction of the left ventricle.   The patient has a history of Q-wave myocardial infarction, which can cause damage to the heart muscle and lead to impaired heart function. The symptoms of shortness of breath with minor exertion, night attacks of dry cough, a feeling of wheezing in the chest and orthopnea, along with the physical exam findings of acrocyanosis, tachycardia, and heart murmurs, suggest heart failure.  

Echocardiography is a common diagnostic test to evaluate cardiac structure and function. The findings of dilatation of the left ventricle, thinning and dyskinesia of the interventricular septum, and reduced ejection fraction (EF) of 39% indicate impaired left ventricular systolic function.

EF is a measure of the percentage of blood that is ejected from the left ventricle with each heartbeat, and a reduced EF indicates reduced pumping ability of the heart.  

Therefore, the correct answer is E, systolic dysfunction of the left ventricle. conditions of a larger group of individuals, such as students in a medical high school, during a training period.


5. A 52-year-old patient complained of sudden weakness and numbness in the left limbs, difficulty in walking. Objectively, left-sided hemihypesthesia and mild hemiparesis were noted. After 4 hours, the patient’s condition normalized, focal symptoms regressed, the patient was able to walk normally. Blood pressure – 120/80 mm Hg. What is the most likely diagnosis? Select one:

A. Transient ischemic attack

B. Associated migraine

C. Hypertensive crisis

D. Ischemic stroke

E. Hemorrhagic stroke


answer: Transient ischemic attack

Explanation

The most likely diagnosis for the patient’s symptoms is a transient ischemic attack (TIA).   A TIA is a temporary episode of neurological dysfunction caused by a temporary disruption of blood flow to a part of the brain. TIAs typically last for a few minutes to a few hours and can cause symptoms similar to those of a stroke, such as sudden weakness, numbness, or tingling on one side of the body, difficulty speaking or understanding speech, and difficulty walking.  

In this case, the patient complained of sudden weakness and numbness in the left limbs, and left-sided hemihypesthesia and mild hemiparesis were noted on examination. However, after 4 hours, the patient’s symptoms resolved, which is characteristic of a TIA.   The patient’s blood pressure was within normal limits, making hypertensive crisis less likely.

Associated migraine can cause neurological symptoms, but the sudden onset of symptoms and their resolution after 4 hours make TIA more likely. Ischemic stroke and hemorrhagic stroke can also cause similar symptoms, but the resolution of symptoms within 4 hours suggests a TIA rather than a stroke.   Therefore, the correct answer is A, transient ischemic attack.  


6. A 22-year-old woman in labor had continuous, very painful cramps after her water broke. Objectively: the dimensions of the pelvis are 25- 28-31-21 cm, the weight of the fetus is 4200 g. The uterus is constantly in tone, the contraction ring is at the level of the navel. The lower segment of the uterus is painful. During vaginal examination: the cervix is fully open, there is no fetal bladder, the fetal head fills the terminal line, the sagittal suture is straight. Fetal heart rate – 136/min. What tactics of the doctor will be most appropriate? Select one:

A. Exclude labor and perform a caesarean section

B. Perform a fertility-destructive operation

C. Apply obstetric forceps

D. Carry out vacuum extraction of the fetus

E. Make a combined rotation of the fetus on the leg, followed by its extraction


Answer: Exclude labor and perform a caesarean section

Explanation

The most appropriate tactic for the doctor in this case would be to exclude labor and perform a caesarean section.   The patient is experiencing continuous, painful uterine contractions after her water has broken, and the dimensions of her pelvis are not favorable for vaginal delivery of a fetus weighing 4200 g.

The fetal head has filled the terminal line, and there is no fetal bladder, indicating that the fetal head is engaged in the pelvis. These findings suggest that the patient is in active labor, but vaginal delivery may be difficult or impossible.   In such cases, a caesarean section is the most appropriate option to ensure a safe delivery for both the mother and the fetus.

Obstetric forceps and vacuum extraction are not recommended in this case due to the large size of the fetus and the unfavorable pelvic dimensions, which can increase the risk of injury to the mother and the fetus during delivery.   A fertility-destructive operation is not indicated in this case, as the patient is in labor and a caesarean section is a safer option for delivering the fetus.   Therefore, the correct answer is A, exclude labor and perform a caesarean section.  


7. a caesarean section A 12-year-old boy came to the clinic with complaints of palpitations, interruptions and pain in the heart, shortness of breath. These symptoms most often occurred after physical exertion and ended with syncopal states. Later, the doctor diagnosed hypertrophic obstructive cardiomyopathy. What drug is contraindicated in the treatment of this pathology? Select one:

A. Verapamil

B. Enalapril

C. Digoxin

D. Bisoprolol

E. –


Answer: Digoxin

Explanation

The drug that is contraindicated in the treatment of hypertrophic obstructive cardiomyopathy (HOCM) is digoxin.   HOCM is a type of cardiomyopathy characterized by abnormal thickening of the heart muscle, particularly the interventricular septum, which can obstruct blood flow from the left ventricle to the aorta.

Symptoms of HOCM may include palpitations, chest pain, shortness of breath, and syncope, especially during or after physical exertion.   Digoxin is a cardiac glycoside that is commonly used in the treatment of heart failure and certain arrhythmias. However, it is contraindicated in patients with HOCM because it can increase the obstruction of blood flow through the left ventricular outflow tract, exacerbating symptoms such as chest pain, syncope, and shortness of breath.  

Other medications commonly used in the treatment of HOCM include beta-blockers such as bisoprolol, calcium channel blockers such as verapamil, and ACE inhibitors such as enalapril. These medications can help improve symptoms and reduce the risk of complications in patients with HOCM.   Therefore, the correct answer is C, digoxin.    

E. Paronychia: Paronychia refers to an infection of the skin around the nail. It typically presents with localized redness, swelling, and tenderness near the nail, but it does not involve the flexor tendon sheath or ligament projection as described in the case.   Therefore, based on the symptoms described, the most likely diagnosis is A. Thecal whitlow (ligament panaritium). However, it is important to consult a healthcare professional for an accurate diagnosis and appropriate treatment.


8. The patient is 54 years old. During the year, he was treated for an undifferentiated form of lung cancer. Currently, a feeling of heaviness and pain in the right hypochondrium have joined. What diagnostic method will be the most informative in this case? Select one:

A. Computed tomography of abdominal organs

B. Ultrasound biolocation with targeted puncture biopsy

C. Vasography of the liver

D. Radioisotope hepatography

E. Diagnostic laparoscopy


Answer: Computed tomography of the abdominal cavity

Explanation

The most informative diagnostic method in this case would be computed tomography (CT) of the abdominal cavity.   The patient has a history of undifferentiated lung cancer and is now experiencing new symptoms of pain and heaviness in the right hypochondrium, which may indicate possible liver involvement by the cancer.

CT scan of the abdominal cavity is a non-invasive imaging technique that can provide detailed information about the liver, as well as other abdominal organs. It can detect the presence of any abnormal masses or lesions in the liver, which may be indicative of metastases from the lung cancer.  

Ultrasound biolocation with targeted puncture biopsy is a useful diagnostic method to confirm the presence of liver metastases and obtain a tissue sample for histopathological examination, but it may not be as informative as CT in detecting small lesions or determining the extent of involvement.  

Vasography of the liver and radioisotope hepatography are less commonly used diagnostic methods for liver metastases, and may not be as sensitive or specific as CT.   Diagnostic laparoscopy is a more invasive diagnostic method that involves inserting a laparoscope into the abdomen to directly visualize the organs and obtain tissue samples.   It may be useful in cases where other diagnostic methods have been inconclusive, but it is not the first-line diagnostic method in this case.  

Therefore, the correct answer is A, computed tomography of the abdominal cavity.     Based on the given information, the milk batch does not appear to have any critical issues regarding fat content and specific density.   However, the presence of microbial activity indicated by the reductase probe result raises concerns about milk quality and potential bacterial contamination.  

In this situation, it is advisable to inform customers about the milk quality. Transparency is important, and customers have the right to be aware of any potential issues.   By providing them with this information, they can make an informed decision about using the milk for their purposes. It is important to maintain consumer trust and ensure their safety.   Option A, selling the milk but informing customers about the milk quality, would be the most responsible and ethical approach in this case.


9. A 16-year-old boy has multiple comedones, follicular nodules, and pustules periodically appear against the background of increased oiliness of the skin of the face, upper back, front surface of the chest. There are no subjective feelings. What is the most likely diagnosis? Select one:

A. Pink eels

B. Vulgar sycosis

C. Furunculosis

D. Common acne

E. Secondary syphilis


Answer: Common acne

Explanation

The most likely diagnosis for the 16-year-old boy with multiple comedones, follicular nodules, and pustules on the face, upper back, and front surface of the chest is common acne.   Acne is a common skin condition that occurs when hair follicles become clogged with oil and dead skin cells, leading to the formation of comedones, papules, pustules, nodules, or cysts.  

Acne can occur on the face, upper back, and chest, and is more common during adolescence due to hormonal changes that increase sebum production.   Pink eels, or rosacea, is a chronic skin condition that causes redness, flushing, and small, red, pus-filled bumps on the face, but it typically affects adults over the age of 30 and is usually not associated with comedones.   Vulgar sycosis, or barber’s itch, is a bacterial infection of the hair follicles that typically affects the beard and neck area, causing pustules and crusted sores.  

Furunculosis is a skin infection that occurs when a hair follicle becomes infected with bacteria, causing a painful, red, swollen bump or boil.   Secondary syphilis is a sexually transmitted infection that can cause skin rashes, including red or pink, non-itchy, scaly patches on the palms and soles of the feet, but it typically presents with other symptoms such as fever, fatigue, and swollen lymph nodes.   Therefore, the most likely diagnosis for the 16-year-old boy is common acne (D).  


10. A study of the relationship between body temperature and pulse rate was conducted on a representative population of flu patients. The calculated correlation coefficient is +0.5. What is the most accurate characteristic of the strength and directionality of the connection between the signs being studied? Select one:

A. Average inverse

B. Weak straight

C. Medium straight

D. Strong straight

E. –


Answer: Average straight line

Explanation

The most accurate characteristic of the strength and directionality of the connection between body temperature and pulse rate, based on a calculated correlation coefficient of +0.5, is a moderate positive correlation.   The correlation coefficient measures the strength and direction of the linear relationship between two variables.

The value of the correlation coefficient ranges from -1 to +1, with -1 indicating a perfect negative correlation,   +1 indicating a perfect positive correlation, and 0 indicating no correlation. A positive correlation means that as one variable increases, the other variable also tends to increase, while a negative correlation means that as one variable increases, the other variable tends to decrease.  

A correlation coefficient of +0.5 indicates a moderate positive correlation, meaning that there is a tendency for body temperature and pulse rate to increase together, but not necessarily in a perfectly linear fashion.  

This suggests that there is some relationship between the two variables, but it is not a strong or perfect relationship.   Therefore, the most accurate characteristic of the strength and directionality of the connection between the signs being studied is a moderate positive correlation, which is option C, medium straight.  


11. A 39-year-old patient complains of dizziness, rapid fatigue. Three days ago, he was operated on for an active duodenal ulcer complicated by bleeding. Operation – suturing of a bleeding ulcer. Objectively: the skin is pale. Blood pressure – 100/60 mm Hg, pulse – 98/min. In the blood: erythrocytes – 2.8 10 /l, Hb – 76 g/l, CP – 0.8, reticulocytes – 8%, platelets – 320 10 l, leukocytes – 9.0 G/l, erythrocyte sedimentation rate – 20 mm/h. What is the most likely diagnosis? Select one:

A. Hemolytic anemia

B. Acute posthemorrhagic anemia

C. Hypoplastic anemia

D. Neurocirculatory dystonia

E. B₁₂-deficiency anemia


Answer: Acute posthemorrhagic anemia

Explanation

The most likely diagnosis for the 39-year-old patient with dizziness, rapid fatigue, and a history of bleeding duodenal ulcer is acute posthemorrhagic anemia.   The patient’s blood pressure is low at 100/60 mm Hg, and the blood tests show a low hemoglobin level of 76 g/L and low erythrocyte count of 2.8 x 10^12/L, which are consistent with an acute loss of blood.

The patient’s pale skin and symptoms of dizziness and fatigue are also indicative of anemia.   Hemolytic anemia, hypoplastic anemia, and B12-deficiency anemia are all types of anemia, but they are not consistent with the patient’s history of bleeding duodenal ulcer and recent surgery.  

Neurocirculatory dystonia is a condition characterized by a variety of symptoms, including dizziness, fatigue, and low blood pressure, but it is not associated with anemia or a recent history of bleeding duodenal ulcer.   Therefore, the most likely diagnosis for the patient is acute posthemorrhagic anemia, which is option B.  


12. A 23-year-old man turned to the doctor with complaints of facial swelling, headaches, dizziness, decreased urine output, and brown urine. The mentioned complaints appeared after the transferred pharyngitis. During the physical examination, body temperature is 37.4°C, blood pressure is 170/110 mm Hg, pulse is 86/min., swelling on the face, pale skin. On auscultation, the heart sounds are muffled, the accent of the II tone over the aorta. What changes are most likely to be observed in the general urinalysis? Select one:

A. Erythrocyturia, proteinuria, hyaline cylinders

B. Erythrocyturia, oxalaturia, erythrocyte cylinders

C. Leukocyturia, proteinuria, epithelial cylinders

D. Erythrocyturia, leukocyturia, leukocyte cylinders

E. Hemoglobinuria, bilirubinuria, granular cylinders


Answer: Erythrocyturia, proteinuria, hyaline cylinders

Explanation

The most likely diagnosis for the 39-year-old patient with dizziness, rapid fatigue, and a history of bleeding duodenal ulcer is acute posthemorrhagic anemia.   The patient’s blood pressure is low at 100/60 mm Hg, and the blood tests show a low hemoglobin level of 76 g/L and low erythrocyte count of 2.8 x 10^12/L, which are consistent with an acute loss of blood.

The patient’s pale skin and symptoms of dizziness and fatigue are also indicative of anemia.   Hemolytic anemia, hypoplastic anemia, and B12-deficiency anemia are all types of anemia, but they are not consistent with the patient’s history of bleeding duodenal ulcer and recent surgery.  

Neurocirculatory dystonia is a condition characterized by a variety of symptoms, including dizziness, fatigue, and low blood pressure, but it is not associated with anemia or a recent history of bleeding duodenal ulcer.   Therefore, the most likely diagnosis for the patient is acute posthemorrhagic anemia, which is option B.  


13. A woman, 6-7 weeks pregnant, turned to a doctor for women’s consultation. From the anamnesis, it was found that he has been suffering from diabetes in a severe form since the age of 15. According to the oculist’s conclusion, retinopathy of the II century. Had 2 pregnancies that ended in stillbirth. Blood sugar content – 15 mmol/l. During a bimanual examination, it was established that the uterus is enlarged (as in pregnancy, up to 7 weeks). What are the doctor’s tactics? Select one:

A. Genetic consultation

B. Consultation of an endocrinologist

C. Termination of pregnancy is indicated

D. Hospitalization in critical periods of pregnancy

E. Prolongation of pregnancy under blood sugar control


Answer: Termination of pregnancy is indicated

Explanation

In this case, termination of pregnancy is indicated.The patient has a history of severe diabetes with retinopathy, and has previously experienced two stillbirths, likely due to complications related to her diabetes. Her blood sugar level is also high at 15 mmol/L, which indicates poor glycemic control.

These factors increase the risk of pregnancy complications, including fetal malformations, preterm labor, and stillbirth.Given the patient’s medical history and current condition, termination of pregnancy is the most appropriate course of action to prevent further complications and risks to both the mother and fetus.

Genetic consultation and consultation with an endocrinologist may be helpful in managing the patient’s medical conditions, but they do not address the immediate risk to the current pregnancy.

Hospitalization in critical periods of pregnancy and prolongation of pregnancy under blood sugar control may be considered in some cases, but given the patient’s history and current condition, termination of pregnancy is the most appropriate option.Therefore, the correct answer is C, termination of pregnancy is indicated.


14. The parents of a 2-year-old boy approached the polyclinic with complaints about the absence of the right testicle in the child’s scrotum. During the examination, hypoplasia of the right half of the valve is revealed, the testicle is absent. It is reduced in size, palpable along the course of the inguinal canal, but does not descend into the portico. What is the most likely diagnosis? Select one:

A. Right-sided cryptorchidism, inguinal form

B. Left-sided monarchism

C. Retraction of the right testicle (pseudocryptorchism)

D. Ectopia of the right testicle, pubic form

E. Right-sided cryptorchidism, ventral form


Answer: Right-sided cryptorchidism, inguinal form

Explanation

The most likely diagnosis for the 2-year-old boy with absence of the right testicle in the scrotum, hypoplasia of the right half of the valve, and a small palpable testicle along the course of the inguinal canal is right-sided cryptorchidism, inguinal form.

Cryptorchidism is a condition where one or both testicles fail to descend into the scrotum during fetal development. In the inguinal form of cryptorchidism, the testicle remains in the inguinal canal and does not descend into the scrotum.Left-sided monarchism, or monorchidism, is a condition where only one testicle is present, but in this case, the patient has a small palpable testicle along the course of the inguinal canal.

Retraction of the right testicle, or pseudocryptorchism, is a condition where the testicle can be felt in the scrotum, but appears to be absent due to retraction.Ectopia of the right testicle, pubic form, is a rare condition where the testicle is located outside the scrotum, but in the pubic region, and is not consistent with the patient’s presentation.

Therefore, the most likely diagnosis for the 2-year-old boy is right-sided cryptorchidism, inguinal form, which is option A


15. The child is 8 years old. Complaints about body temperature of 39.3°C, headache, vomiting. 1st day of illness. The skin is clean. The throat is hyperemic. The tongue is coated. Conjunctivitis. Scleritis. Weakly expressed rigidity of the occipital muscles. Positive Kernig sign on the right. CSF: cytosis – 340/µl, lymphocytes – 87%, neutrophils – 13%, protein – 140 mg/l. What would be the most likely diagnosis? Select one:

A. Arachnoiditis

B. Brain tumor

C. Serous meningitis

D. Purulent meningitis

E. Neurotoxicosis


Answer is: Serous meningitis

Explanation

The most likely diagnosis for the 8-year-old child with a body temperature of 39.3°C, headache, vomiting, hyperemic throat, coated tongue, conjunctivitis, scleritis, weakly expressed rigidity of the occipital muscles, positive Kernig sign on the right, and cerebrospinal fluid (CSF) findings of lymphocytic pleocytosis and elevated protein level is serous meningitis.  

Serous meningitis, also known as aseptic meningitis, is a type of meningitis that is typically caused by a viral infection and is characterized by inflammation of the meninges, the membranes that surround the brain and spinal cord. The symptoms of serous meningitis include fever, headache, vomiting, neck stiffness, and photophobia, and can sometimes be accompanied by conjunctivitis, scleritis, and other symptoms.  

Arachnoiditis, brain tumor, and neurotoxicosis are less likely diagnoses based on the symptoms and CSF findings presented. Arachnoiditis is a rare condition characterized by inflammation and scarring of the arachnoid membrane, which can cause chronic pain and neurological symptoms. Brain tumor is a possibility, but typically presents with more specific symptoms and findings on imaging studies. Neurotoxicosis refers to poisoning by a toxic substance, but the symptoms and CSF findings presented are not consistent with this diagnosis.  

Purulent meningitis is a type of meningitis caused by a bacterial infection, and typically presents with more severe symptoms, including high fever, severe headache, seizures, and altered mental status. The CSF findings in purulent meningitis also typically show a predominance of neutrophils, rather than lymphocytes.   Therefore, the most likely diagnosis for the 8-year-old child is serous meningitis, which is option C.  


16. A 27-year-old electrician was electrocuted by touching bare electrical wiring with his hand. Blood circulation and breathing stopped. Resuscitation measures ensured the recovery of cardiac activity after 5 minutes. Which of the complications is most likely a few hours or even days after an electric shock? Select one:

A. Acute liver failure

B. Pulmonary edema

C. Cessation of blood circulation

D. Acute renal failure

E. Stop breathing


Answer: Cessation of blood circulation

Explanation

The most likely complication a few hours or even days after an electric shock in a patient who experienced cardiac arrest and required resuscitation measures is cessation of blood circulation.   Electric shock can cause a range of injuries, including burns, muscle damage, nerve damage, and cardiac arrest. In this case, the patient required resuscitation measures to restore cardiac activity, which indicates that the electric shock was severe enough to cause cardiac arrest.  

Cessation of blood circulation is a potential complication that can occur after an electric shock, particularly if the shock caused damage to the heart or blood vessels. This could lead to decreased cardiac output and blood pressure, which can result in organ failure or other complications.   Acute liver failure, pulmonary edema, and acute renal failure are also potential complications of electric shock, but they are less likely in this case given the presentation of cardiac arrest and the need for resuscitation measures.  

Stop breathing is also a potential complication of electric shock, but in this case, the patient’s breathing was not mentioned to have stopped, and the focus was on the resuscitation measures needed to restore cardiac activity.   Therefore, the most likely complication a few hours or even days after an electric shock in this case is cessation of blood circulation, which is option C.  


17. A 20-year-old patient with symptoms of polyarthritis came to the rheumatology department. Redness in the form of a “butterfly” on the skin of the face. In urine: protein – 4.8 g/l. The doctor assumed that the patient had systemic lupus erythematosus. What additional research method is the most informative for verifying the diagnosis? Select one:

A. Blood analysis for antinuclear antibodies

B. Immunological examination of blood

C. General blood test

D. Blood analysis for rheumatoid factor

E. Blood analysis for LE cells


Answer: Blood test for antinuclear antibodies

Explanation

The most informative additional research method for verifying the diagnosis of systemic lupus erythematosus (SLE) in the 20-year-old patient with symptoms of polyarthritis, redness in the form of a “butterfly” on the skin of the face, and proteinuria is a blood test for antinuclear antibodies (ANA).  

SLE is an autoimmune disease that can affect multiple organs and tissues, and is characterized by the presence of autoantibodies, including ANA. ANA are present in up to 95% of patients with SLE, and are considered a key diagnostic criterion.  

Immunological examination of blood, blood analysis for rheumatoid factor, and blood analysis for LE cells are less specific and sensitive tests for SLE, and may be positive in other autoimmune diseases or conditions. A general blood test may show non-specific signs of inflammation, but is not diagnostic for SLE.  

Therefore, the most informative additional research method for verifying the diagnosis of SLE in this patient is a blood test for antinuclear antibodies (ANA), which is option A.


18. During resuscitation measures, a 77-year-old patient cannot perform chest compressions to a depth of 5-6 cm due to chest stiffness. What will be your actions? Select one:

A. Place the pressure point in the direction of the xiphoid process

B. Increase the frequency of clicks to 150 times per minute

C. Continue pressing with the previous force

D. Increase the force of pressing without taking into account the risk of rib fracture

E. Reduce the pressure to prevent rib and sternum fractures


Answer: Increase the strength of the presses, regardless of the risk of rib fractures

Explanation

If a 77-year-old patient cannot perform chest compressions to a depth of 5-6 cm due to chest stiffness during resuscitation measures, the appropriate action would be to increase the strength of the compressions, regardless of the risk of rib fractures. Therefore, the correct answer is D.

During cardiopulmonary resuscitation (CPR), chest compressions are performed to maintain blood circulation and oxygenation of vital organs. The recommended depth of chest compressions is 5-6 cm, which can be difficult to achieve in older adults due to chest stiffness or other factors.In this case, increasing the force of chest compressions is necessary to ensure adequate blood flow and oxygenation to vital organs, even if it carries a risk of rib or sternum fractures.

The risk of rib fractures during chest compressions is relatively low, and the benefit of maintaining blood flow and oxygenation to vital organs outweighs this risk.Placing the pressure point in the direction of the xiphoid process or increasing the frequency of compressions may not be effective in achieving adequate depth of compressions.

Additionally, reducing the pressure to prevent rib and sternum fractures may compromise the effectiveness of CPR.Therefore, increasing the strength of chest compressions, regardless of the risk of rib fractures, is the appropriate action in this case.


19. A 35-year-old patient turned to the doctor with complaints about the presence of a neoplasm on the skin of her right lower leg. Objectively: a nevus of an irregular shape with a diameter of 1 cm, without clear boundaries, with a dry surface, dark brown in color. The diagnosis was established: probable melanoma of the skin of the right lower leg. What is the method of verifying the diagnosis of skin melanoma? Select one:

A. Positron emission tomography – computer tomography (PET-CT)

B. Excisional biopsy

C. Smudge-imprint

D. Ultrasound examination (ultrasound)

E. Incisional biopsy


Answer: Excisional biopsy

Explanation

The method of verifying the diagnosis of skin melanoma in a patient with a nevus of an irregular shape with a diameter of 1 cm, without clear boundaries, with a dry surface, dark brown in color on the skin of the right lower leg is excisional biopsy. Melanoma is a type of skin cancer that can be difficult to diagnose based on clinical examination alone.

A definitive diagnosis of melanoma requires a biopsy of the suspicious lesion. An excisional biopsy involves removing the entire suspicious lesion, along with a margin of normal tissue around it, for examination under the microscope. This is the preferred method of biopsy for suspected melanoma, as it provides the most accurate diagnosis and allows for assessment of the depth of invasion and other important features of the tumor. 

PET-CT, ultrasound, and smudge-imprint are not typically used for the diagnosis of skin melanoma. PET-CT may be useful in staging and monitoring the progression of melanoma, but is not a diagnostic tool. Ultrasound may be used to determine the depth of invasion of a melanoma, but is not typically used for diagnosis.

Smudge-imprint, also known as touch preparation, is a rapid diagnostic technique that involves pressing a glass slide onto the cut surface of a lesion and examining the cells under the microscope, but it is not as definitive as an excisional biopsy. 

Incisional biopsy, which involves removing a portion of the suspicious lesion for examination, may be used in some cases of melanoma, but is not as accurate as an excisional biopsy. Therefore, the method of verifying the diagnosis of skin melanoma in this patient is excisional biopsy, which is option B. 


20. A girl weighing 3100 g was born from her third pregnancy at home. On the 3rd day, the mother noticed small hemorrhages in the umbilical wound, as well as “red spots” on the sclera of the eyes. The mother was not registered for pregnancy in the prenatal period. Feeds the baby with breast milk as needed. During physical examination, subconjunctival hemorrhages. During the laboratory examination, the hemoglobin concentration is 155 g/l, the mother’s blood group is B (III) Rh-positive, the child’s blood group is A (II) Rh-negative. What tactic to prevent the progression of this condition in a child would be the most appropriate? Select one:

A. Enter vitamin K

B. Administration of human anti-Rhesus Rh0 (D) immunoglobulin

C. Transition to artificial feeding with ironcontaining mixtures

D. Transfusion of erythrocyte mass

E. Prescribe oral iron preparations


Answer: Enter vitamin K

Explanation

The most appropriate tactic to prevent the progression of the condition in the newborn girl with small hemorrhages in the umbilical wound and subconjunctival hemorrhages is to administer vitamin K.Vitamin K is essential for blood clotting, and newborns are at risk for bleeding disorders due to low levels of vitamin K at birth.Breast milk contains low levels of vitamin K, and infants who are exclusively breastfed may be at increased risk for bleeding disorders.

In this case, the newborn girl was born at home and was not registered for pregnancy in the prenatal period, which increases the likelihood that the mother did not receive adequate prenatal care or counseling on the importance of vitamin K supplementation for the newborn.Administration of vitamin K is a safe and effective way to prevent bleeding disorders in newborns, and is recommended by the American Academy of Pediatrics and other medical organizations.

Vitamin K can be administered orally or by injection, and the most appropriate route of administration can be determined by the healthcare provider based on the infant’s clinical status and other factors.Administration of human anti-Rh(D) immunoglobulin, transfusion of erythrocyte mass, or prescription of oral iron preparations are not indicated in this case, as there is no evidence of Rh incompatibility or anemia. Transition to artificial feeding with iron-containing mixtures may be necessary in some cases of iron deficiency anemia, but is not indicated in this case.

Therefore, the most appropriate tactic to prevent the progression of the condition in this newborn girl is to administer vitamin K, which is option A.affects the elimination of waste products from the body. The absence of edema and the findings of single red blood cells (RBC) and hyaline casts in the urine are consistent with chronic nephritis.

Feochromocytoma (option B) is a tumor of the adrenal gland that can cause high blood pressure, but it doesn’t explain the other symptoms or laboratory findings in this case.Hypertensive illness of the II degree (option C) is a possibility given the elevated blood pressure, but it does not fully explain the presence of proteinuria and other kidney-related findings.

Nephrotic syndrome (option D) typically presents with significant proteinuria, edema, and low levels of protein in the blood, which are not prominent in this case.Stenosis of the kidney artery (option E) is a narrowing of the renal artery, which can lead to high blood pressure, but it does not explain the other symptoms or laboratory findings.In conclusion, based on the provided information, the most appropriate diagnosis is chronic nephritis with violation of kidney function. However, it’s important to consult a healthcare professional for a comprehensive evaluation and accurate diagnosis.


21. An eight-year-old child complains of a long wet cough with the release of a large amount of sputum of a purulent nature with an unpleasant smell, sometimes with an admixture of blood. The general condition is severe, the body temperature is 38.7°C, the skin is pale, perioral cyanosis, fingers in the form of “drumsticks”. Over the lungs: weakened breathing, wet rales of various calibers. X-ray: atelectatic areas. Which of the examinations to clarify the diagnosis will be the most appropriate? Select one:

A. Bronchoscopy

B. Computed tomography with high resolution

C. –

D. Echocardiography

E. X-ray of OGK


Answer: Computed tomography with high resolution

Explanation

In an eight-year-old child with a long wet cough, release of a large amount of purulent sputum with an unpleasant smell, atelectatic areas on X-ray, and a severe general condition with perioral cyanosis and finger clubbing, the most appropriate examination to clarify the diagnosis would be computed tomography (CT) with high resolution. 

The clinical presentation of this child suggests a severe respiratory infection, possibly with a bacterial etiology, which has resulted in atelectasis and consolidation of lung tissue. The wet rales on lung examination also suggest the presence of significant airway obstruction and inflammation. 

CT with high resolution provides detailed images of lung tissue and can help identify the extent and severity of pulmonary involvement, including areas of consolidation, atelectasis, and other abnormalities. This can help guide treatment decisions, such as the need for antibiotics or other interventions.

Bronchoscopy may also be useful in some cases to visualize the airways and obtain samples for culture and sensitivity testing, but it may not be practical or safe in a child with severe respiratory distress. Echocardiography and X-ray of the OGK are unlikely to provide useful information in this case, as the clinical presentation and X-ray findings suggest a primarily pulmonary etiology.

Therefore, the most appropriate examination to clarify the diagnosis in this child would be computed tomography with high resolution, which is option B.Option C (Erosive gastritis) is less likely because erosive gastritis typically does not cause chronic painless ulcers.

Option D (Acute pleurisy) and option E (Acute myocardial infarction, abdominal form) are less likely as they do not explain the patient’s symptoms and history of a stomach ulcer.In conclusion, based on the provided information, the most likely diagnosis is an ulcer of the stomach complicated with bleeding. However, it is important to consult a healthcare professional for a thorough evaluation and accurate diagnosis.


22. A 35-year-old woman complains of heart failure, weight loss, pain in the front of the neck, which started after SARS. Pulse — 110/min, blood pressure — 130/60 mm Hg. Art. The thyroid gland is slightly enlarged, painful on palpation. Laboratory revealed: ESR— 40 mm/h. TSH — 0.05 mU/l, T4 — 34.3 pmol/l. What is the most likely diagnosis?

a. Subacute thyroiditis

b. Hashimoto’s thyroiditis

c. Paroxysmal tachycardia

d. Diffuse toxic goiter

e. Acute thyroiditis


Answer: Subacute thyroiditis

Explanation

The most likely diagnosis in a 35-year-old woman with heart failure, weight loss, pain in the front of the neck, slightly enlarged, and painful thyroid gland on palpation, and laboratory findings of elevated ESR, low TSH, and high T4 after a recent history of SARS is subacute thyroiditis.

Subacute thyroiditis is an inflammatory condition of the thyroid gland that can occur following a viral infection, such as SARS. It typically presents with neck pain, tenderness, and an enlarged, firm, and painful thyroid gland. Subacute thyroiditis can also cause hyperthyroidism, which is characterized by a low TSH and high T4.

The heart failure and weight loss may be related to the hyperthyroidism.Hashimoto’s thyroiditis is an autoimmune disorder that causes chronic inflammation of the thyroid gland, leading to hypothyroidism rather than hyperthyroidism. Paroxysmal tachycardia is a type of arrhythmia that may cause rapid heart rate, but is not typically associated with thyroiditis.

Diffuse toxic goiter, also known as Graves’ disease, is a type of autoimmune hyperthyroidism that may cause an enlarged thyroid gland, but is less likely to cause neck pain and tenderness.Acute thyroiditis is a rare bacterial infection of the thyroid gland that can cause fever, chills, and severe pain in the neck, but is not typically associated with recent viral infections or hyperthyroidism.Therefore, the most likely diagnosis in this patient is subacute thyroiditis, which is option A. 


23. A 7-year-old boy was brought to the doctor with complaints of pronounced total cyanosis with a purple tint, shortness of breath, which is relieved in a squatting position with knees pressed to the abdomen. When examining fingers in the form of “drumsticks” and nails as “watch glasses”, auscultatory blowing noise over the pulmonary artery. During instrumental examination, the vertical axis of the heart is shown on the electrocardiogram. On echocardiography, pulmonary artery stenosis, right ventricular wall hypertrophy, ventricular septal defect, aortic dextroposition. What is the most likely diagnosis? Select one:

A. Ventricular septal defect

B. Stenosis of the opening of the pulmonary artery

C. Transposition of the main vessels

D. The ductus arteriosus is open

E. Tetralogy of Fallot


Answer: Tetrada Fallo

Explanation

The most likely diagnosis in a 7-year-old boy with pronounced total cyanosis, shortness of breath relieved in a squatting position, fingers in the form of “drumsticks,” nails as “watch glasses,” auscultatory blowing noise over the pulmonary artery, vertical axis of the heart on electrocardiogram, and echocardiography findings of pulmonary artery stenosis, right ventricular wall hypertrophy, ventricular septal defect, and aortic dextroposition is Tetralogy of Fallot. 

Tetralogy of Fallot is a congenital heart defect that includes four abnormalities: pulmonary artery stenosis, right ventricular wall hypertrophy, ventricular septal defect, and aortic dextroposition.

These abnormalities result in reduced blood flow to the lungs and decreased oxygenation of the blood, leading to cyanosis and shortness of breath. The characteristic squatting position with knees pressed to the abdomen helps increase blood flow to the lungs and relieve symptoms. 

Ventricular septal defect, stenosis of the opening of the pulmonary artery, and patent ductus arteriosus are also congenital heart defects that can cause cyanosis and shortness of breath, but do not typically present with the characteristic squatting position or the combination of echocardiography findings seen in Tetralogy of Fallot. 

Transposition of the main vessels is a congenital heart defect that involves a reversal of the position of the pulmonary artery and the aorta, which can result in cyanosis, but typically presents in the neonatal period with severe symptoms. Therefore, the most likely diagnosis in this patient is Tetralogy of Fallot, which is option E. 


24. A 14-year-old student came to the family doctor with complaints about scratches on her face caused by a neighbor’s cat 2 days ago. When examined, the scratches are superficial, covered with a scab. Primary surgical treatment of wounds was performed. It is known that the child received all scheduled vaccinations according to the calendar. What will be the next steps? Select one:

A. Administration of anti-rabies vaccine

B. Administration of anti-rabies vaccine + anti-rabies immunoglobulin

C. Observation of the animal

D. Administration of rabies vaccine + tetanus toxoid

E. Administration of anti-rabies immunoglobulin


Answer: Observation of an animal

Explanation

In a 14-year-old student with superficial scratches on the face caused by a neighbor’s cat 2 days ago, who has received all scheduled vaccinations according to the calendar, the next step would be observation of the animal.Rabies is a serious viral infection that can be transmitted through the saliva of infected animals, including cats.

However, the risk of rabies transmission from a superficial scratch is low, especially if the animal is known to be vaccinated against rabies. Additionally, the patient has received all scheduled vaccinations according to the calendar, which includes vaccination against rabies.

According to the Centers for Disease Control and Prevention (CDC) guidelines, if an animal that has potentially exposed a person to rabies is available for observation, the animal should be observed for 10 days to determine if it develops any signs of rabies. If the animal remains healthy during the observation period, then the person is considered not to have been exposed to rabies and no further treatment is necessary.If the animal becomes ill or dies during the observation period or if the animal is unavailable for observation, then post-exposure prophylaxis may be necessary.

In this case, since the scratches are superficial and the patient has received all scheduled vaccinations, no further treatment is necessary at this time.Therefore, the next step in this patient would be observation of the animal, which is option C.Menetrier’s disease (option D) is a rare condition characterized by enlarged rugae in the stomach, excessive production of mucus, and protein loss.

It is not consistent with the symptoms and findings described in the case.Rigid antral gastritis (option E) is not a recognized medical term. It may be a misinterpretation or a non-standard term.Therefore, based on the given information, the most appropriate diagnosis is gastritis of type B, associated with Helicobacter pylori infection.


25. On the shore of the sea you saw a young boy lying on the ground. Goosebumps, sharply cyanotic skin, white foam on the lips. The pulse is frequent, arrhythmic, breathing is not heard in the lungs. What will be the first actions at the pre-hospital stage? Select one:

A. Remove mucus and foam from the mouth and oropharynx, perform artificial respiration

B. Provide a position with raised legs

C. Make a sharp blow to the sternum

D. Rub and warm the patient

E. Leave alone, call an ambulance


Answer: Remove mucus and foam from the mouth and oropharynx, perform artificial respiration

Explanation

The first action that should be taken in a young boy lying on the ground with goosebumps, sharply cyanotic skin, white foam on the lips, frequent arrhythmic pulse, absent lung sounds, and signs of possible drowning is to remove mucus and foam from the mouth and oropharynx and perform artificial respiration, which is option A. 

These signs and symptoms suggest that the boy may have experienced a near-drowning event, which can result in airway obstruction due to the presence of water or foam in the lungs and airways. The first priority in this situation is to ensure that the airway is clear and that the patient is receiving adequate oxygenation. 

The airway should be cleared by carefully removing any water, mucus, or foam from the mouth and oropharynx using gentle suction or a finger sweep. Artificial respiration, such as mouth-to-mouth or bag-mask ventilation, should be initiated immediately to provide oxygen to the lungs and restore normal breathing. Providing a position with raised legs, making a sharp blow to the sternum, or rubbing and warming the patient are not appropriate interventions in this situation.

Leaving the patient alone and calling an ambulance without providing immediate care can lead to further deterioration of the patient’s condition. Therefore, the first action that should be taken in this patient is to remove mucus and foam from the mouth and oropharynx and perform artificial respiration, which is option A.  

The first action that should be taken in a young boy lying on the ground with goosebumps, sharply cyanotic skin, white foam on the lips, frequent arrhythmic pulse, absent lung sounds, and signs of possible drowning is to remove mucus and foam from the mouth and oropharynx and perform artificial respiration, which is option A. These signs and symptoms suggest that the boy may have experienced a near-drowning event, which can result in airway obstruction due to the presence of water or foam in the lungs and airways.

The first priority in this situation is to ensure that the airway is clear and that the patient is receiving adequate oxygenation. The airway should be cleared by carefully removing any water, mucus, or foam from the mouth and oropharynx using gentle suction or a finger sweep. Artificial respiration, such as mouth-to-mouth or bag-mask ventilation, should be initiated immediately to provide oxygen to the lungs and restore normal breathing. 

Providing a position with raised legs, making a sharp blow to the sternum, or rubbing and warming the patient are not appropriate interventions in this situation. Leaving the patient alone and calling an ambulance without providing immediate care can lead to further deterioration of the patient’s condition. Therefore, the first action that should be taken in this patient is to remove mucus and foam from the mouth and oropharynx and perform artificial respiration, which is option A.


26. During the last 6 months, the 23-year-old patient became withdrawn, avoided communication with others, secluded himself, and talked to himself. During the conversation with the psychiatrist, at first he hid his feelings, but then he told about special, unreal voices contained in himself (“nesting in the brain”). What are these perception disorders? Select one:

A. Pseudohallucinations

B. Illusions

C. Metamorphopsia

D. Hallucinations are real

E. Derealization


The answer is: Pseudohallucinations

 Explanation

The perception disorders described in the scenario for the 23-year-old patient are pseudohallucinations. Pseudohallucinations are perceptions that are similar to true hallucinations, but the individual recognizes that the perception is not real and is coming from within their own mind rather than from an external source.

In pseudohallucinations, the individual may describe hearing voices or seeing images that are not present in the external environment, but they are aware that these perceptions are not real. In contrast, with true hallucinations, the individual believes that the perceptions are real and coming from an external source, even if they are not actually present in the environment. The patient in the scenario describes special, unreal voices that are contained within himself, which is characteristic of pseudohallucinations.

The patient’s withdrawal, avoidance of communication with others, and seclusion are also consistent with psychotic symptoms.Illusions are misperceptions of real external stimuli, metamorphopsia is a type of visual distortion, hallucinations are perceptions that are believed to be real and coming from an external source, and derealization is a sense of unreality or detachment from the environment.Therefore, the perception disorders described in this patient are pseudohallucinations, which is option A. 

The perception disorders described in the scenario for the 23-year-old patient are pseudohallucinations.Pseudohallucinations are perceptions that are similar to true hallucinations, but the individual recognizes that the perception is not real and is coming from within their own mind rather than from an external source. In pseudohallucinations, the individual may describe hearing voices or seeing images that are not present in the external environment, but they are aware that these perceptions are not real.

In contrast, with true hallucinations, the individual believes that the perceptions are real and coming from an external source, even if they are not actually present in the environment.The patient in the scenario describes special, unreal voices that are contained within himself, which is characteristic of pseudohallucinations. The patient’s withdrawal, avoidance of communication with others, and seclusion are also consistent with psychotic symptoms.

Illusions are misperceptions of real external stimuli, metamorphopsia is a type of visual distortion, hallucinations are perceptions that are believed to be real and coming from an external source, and derealization is a sense of unreality or detachment from the environment.Therefore, the perception disorders described in this patient are pseudohallucinations, which is option A. 


27. A 35-year-old patient was hospitalized with attacks of fever, which were accompanied by sweating and recurred every 4 days. Objectively: subicteric sclera, pale skin, liver enlargement by 2 cm and its compaction, spleen enlargement by 5 cm. What research will allow to verify the diagnosis? Select one:

A. Research on the activity of ALT and AST

B. Microscopy of a thick drop and blood smear for malarial plasmodium

C. Biochemical analysis of blood

D. Serological examination of blood

E. Determination of the level of bilirubin in blood serum


Answer : Microscopy of a thick drop and blood smear for malarial plasmodium

 Explanation

The clinical presentation of the 35-year-old patient with attacks of fever, sweating, subicteric sclera, pale skin, liver enlargement, and spleen enlargement is suggestive of malaria, which is a parasitic infection transmitted by the bite of infected Anopheles mosquitoes. 

The most appropriate diagnostic test for malaria is microscopy of a thick drop and blood smear for malarial plasmodium, which is option B. This test involves examining a drop of the patient’s blood under a microscope to identify the presence of malarial parasites. This test has high sensitivity and specificity for the diagnosis of malaria. 

Research on the activity of ALT and AST, biochemical analysis of blood, determination of the level of bilirubin in blood serum, and serological examination of blood may be useful in evaluating liver function and ruling out other causes of fever and hepatosplenomegaly, but they are not specific for the diagnosis of malaria.Therefore, the most appropriate research to verify the diagnosis in this patient is microscopy of a thick drop and blood smear for malarial plasmodium, which is option B.

The clinical presentation of the 35-year-old patient with attacks of fever, sweating, subicteric sclera, pale skin, liver enlargement, and spleen enlargement is suggestive of malaria, which is a parasitic infection transmitted by the bite of infected Anopheles mosquitoes.The most appropriate diagnostic test for malaria is microscopy of a thick drop and blood smear for malarial plasmodium, which is option B. This test involves examining a drop of the patient’s blood under a microscope to identify the presence of malarial parasites.

This test has high sensitivity and specificity for the diagnosis of malaria.Research on the activity of ALT and AST, biochemical analysis of blood, determination of the level of bilirubin in blood serum, and serological examination of blood may be useful in evaluating liver function and ruling out other causes of fever and hepatosplenomegaly, but they are not specific for the diagnosis of malaria.Therefore, the most appropriate research to verify the diagnosis in this patient is microscopy of a thick drop and blood smear for malarial plasmodium, which is option B.


 28. A 49-year-old woman consulted a doctor with complaints of headache, hot flushes to the head and neck, increased sweating, palpitations, increased blood pressure up to 170/100 mm Hg, irritability, insomnia, tearfulness, memory loss, infrequent scanty menstruation, an increase in body weight by 5 kg during the last six months. What is the most likely diagnosis? Select one:

A. Somatoform disorder

B. Premenstrual syndrome

C. Climacteric syndrome

D. Post-castration syndrome

E. Arterial hypertension


Answer: Climacteric syndrome

Explanation

The most likely diagnosis for the 49-year-old woman with complaints of headache, hot flushes to the head and neck, increased sweating, palpitations, increased blood pressure, irritability, insomnia, tearfulness, memory loss, infrequent scanty menstruation, and an increase in body weight during the last six months is climacteric syndrome, also known as perimenopause or menopausal transition.

Climacteric syndrome is a collection of symptoms that occur during the transition to menopause, which is the cessation of menstrual periods due to the cessation of ovarian function. The symptoms of climacteric syndrome are caused by changes in hormonal levels, particularly a decline in estrogen production, and can include hot flushes, night sweats, palpitations, mood changes, cognitive changes, and changes in menstrual patterns.The patient’s age, symptoms, and menstrual pattern are consistent with the diagnosis of climacteric syndrome.

While arterial hypertension can cause some of the symptoms described, the constellation of symptoms in this patient is more consistent with climacteric syndrome.Somatoform disorder and premenstrual syndrome are unlikely diagnoses given the patient’s age and clinical presentation. Post-castration syndrome is not a recognized medical diagnosis.Therefore, the most likely diagnosis for this patient is climacteric syndrome, which is option C.


29. A 36-year-old woman consulted a doctor with complaints of frequent urination, pain in the left side, nausea, and high temperature. During physical examination, body temperature is 39.4°C, blood pressure is 120/80 mm Hg, pulse is 89/min, respiratory rate is 18/min, SpO₂ is 98% in room air. On palpation, soreness in the left costovertebral corner and left side. What would be the most appropriate next step for the doctor in the management of the patient? Select one:

A. Start intravenous ceftriaxone

B. Prescribe trimethoprim-sulfamethoxazole for 10 days

C. Administer nitrofurantoin orally for 10 days

D. Recommend taking a pregnancy test

E. Pass a general urinalysis and perform a bacteriological examination of urine


Answer: Take a general urinalysis and perform a bacteriological examination of the urine

Explanation

The clinical presentation of the 36-year-old woman with frequent urination, pain in the left side, nausea, high temperature, soreness in the left costovertebral corner and left side is suggestive of a urinary tract infection (UTI), specifically pyelonephritis, which is an infection of the kidney.

The most appropriate next step for the doctor in the management of the patient is to take a general urinalysis and perform a bacteriological examination of the urine, which is option E. A urinalysis can confirm the diagnosis of a UTI and identify the causative organism, which is important for guiding appropriate antibiotic therapy.

Starting intravenous ceftriaxone or prescribing trimethoprim-sulfamethoxazole or nitrofurantoin without confirmation of the diagnosis may lead to inappropriate antibiotic use and potential adverse effects. Therefore, it is important to confirm the diagnosis of UTI before initiating treatment.

Recommending a pregnancy test may be appropriate if the patient is of childbearing age and there is a possibility of pregnancy, but it is not relevant to the management of the current presentation.

Therefore, the most appropriate next step for the doctor in the management of the patient is to take a general urinalysis and perform a bacteriological examination of the urine, which is option E. breathing and muscle cramps.

It’s important to note that a definitive diagnosis and appropriate medical intervention should be made by a healthcare professional based on a comprehensive evaluation of the patient’s history, physical examination, and laboratory tests.


30.A 3-year-old child has been sick for 3 days. Fever, cough, runny nose, conjunctivitis, photophobia, a whitish dot rash on the mucous membrane of the gums and the inner surface of the cheeks and spotted enanthema of the hard and soft palate are observed. What disease can be diagnosed? Select one:

A. Enterovirus infection

B. Acute respiratory viral infection

C. Herpetic infection

D. Infectious mononucleosis

E. Enter


Answer: Кір

Explanation

The symptoms described in the scenario, including fever, cough, runny nose, conjunctivitis, photophobia, a whitish dot rash on the mucous membrane of the gums and the inner surface of the cheeks, and spotted enanthema of the hard and soft palate, are consistent with the diagnosis of measles (rubella), which is a highly contagious viral infection caused by the measles virus. 

Measles is characterized by a prodromal phase of fever, cough, runny nose, and conjunctivitis, followed by the appearance of the characteristic rash, which typically begins on the face and spreads to the trunk and limbs. The whitish dot rash, known as Koplik’s spots, is a hallmark of measles and appears on the mucous membrane of the gums and the inner surface of the cheeks. Spotted enanthema of the hard and soft palate is also a characteristic feature of measles. 

Enterovirus infection, acute respiratory viral infection, and herpetic infection may present with similar symptoms, but do not typically have the characteristic rash or enanthem associated with measles.Infectious mononucleosis may present with fever, sore throat, and swollen lymph nodes, but does not typically have the characteristic rash or enanthem associated with measles.

Therefore, based on the symptoms described in the scenario, the most likely diagnosis is measles (rubella), which is option E. airways of the lungs, but it does not cause situs inversus.Option D (Laryngotracheomalacia) is unrelated to the symptoms described in the scenario. It refers to the softening of the tissues of the larynx and trachea, which can cause breathing difficulties in infants.

Option E (α-antitrypsin deficiency) is a genetic disorder that primarily affects the liver and lungs, leading to liver disease and early-onset emphysema. It does not cause situs inversus.It’s important to consult with a healthcare professional for a comprehensive evaluation and accurate diagnosis in order to determine the underlying cause of the symptoms.


31. The mother of a 1-year-old child complains of constant intrusive, frequent, unproductive cough, sometimes leading to vomiting. During an objective examination, the patient has rapid breathing, moderate retraction of the lower intercostal muscles, and an increase in the anteroposterior size of the chest. Bronchoobstruction appears during respiratory infections. During the newborn period, he suffered meconium ileus. Choose the primary examination: Select one:

A. Studies of sweat chlorides

B. Research on chlamydia and mycoplasma

C. Genetic testing

D. X-ray of chest organs

E. CT scan of the lungs


Answer: Examination of sweat chlorides

Explanation

The clinical presentation of the 1-year-old child with constant intrusive, frequent, unproductive cough, rapid breathing, moderate retraction of the lower intercostal muscles, and an increase in the anteroposterior size of the chest is suggestive of cystic fibrosis, which is a genetic disorder that affects the respiratory, digestive, and reproductive systems. The most appropriate primary examination for this child is studies of sweat chlorides, which is option A.

The diagnosis of cystic fibrosis is confirmed by the presence of elevated sweat chloride levels, which is a hallmark of the disorder. Sweat testing is a simple, noninvasive test that measures the amount of chloride in sweat and can be done in specialized laboratories. 

Research on chlamydia and mycoplasma, genetic testing, X-ray of chest organs, and CT scan of the lungs are not specific for the diagnosis of cystic fibrosis and are unlikely to provide diagnostic information in this case. Therefore, the most appropriate primary examination for this child is studies of sweat chlorides, which is option A. 


32. The employee was hospitalized for 16 days for pneumonia. What is the procedure for issuing a certificate of incapacity for work in this case? Select one:

A. The attending physician from the day of admission to the hospital for a maximum of 14 days

B. The supervising physician together with the head of the department for the entire period of treatment

C. The attending physician from the day of admission to the hospital for a maximum of 5 days

D. According to the conclusion of the medical and advisory commission for the entire period of treatment

E. The supervising physician from the day of admission to the hospital for a maximum of 10 days


Answer : The supervising physician together with the head of the department for the entire period of treatment

Explanation

In many countries, the procedures for issuing a certificate of incapacity for work may vary, but in general, the supervising physician is responsible for issuing the certificate for a patient who has been hospitalized for an extended period. In this case, since the employee was hospitalized for 16 days for pneumonia, the most appropriate procedure for issuing a certificate of incapacity for work would be for the supervising physician to issue the certificate, together with the head of the department, for the entire period of treatment.

This is option B. The supervising physician is responsible for the overall management of the patient’s care during the hospitalization, and is in the best position to determine the patient’s ability to work based on their medical condition and progress.  

The head of the department may also be involved in the decision-making process, particularly if there are any concerns or complications during the hospitalization. Therefore, the most appropriate procedure for issuing a certificate of incapacity for work for an employee who has been hospitalized for 16 days for pneumonia would be for the supervising physician to issue the certificate, together with the head of the department, for the entire period of treatment, which is option B. 


33. After a hemorrhagic stroke, the patient undergoes a course of massage, physical therapy, and speech therapy to restore speech and movements of the left lower and upper limbs. What type of prevention do these rehabilitation measures belong to? Select one:

A. Public prevention

B. Secondary prevention

C. Primary prevention

D. Tertiary prevention

E. Individual prevention


Answer : Tertiary prevention

Explanation

The rehabilitation measures of massage, physical therapy, and speech therapy after a hemorrhagic stroke are aimed at restoring the patient’s function and preventing further disability.  These measures are part of tertiary prevention, which involves the management and rehabilitation of patients with established disease or disability. 

Tertiary prevention focuses on improving the quality of life of patients with existing health problems and preventing complications and further deterioration.  The goal is to help patients achieve the highest possible level of functioning and independence, and to minimize the impact of their condition on their daily activities. 

In this case, the patient has already had a hemorrhagic stroke, and the rehabilitation measures are aimed at preventing further disability and improving their ability to perform activities of daily living.  Massage, physical therapy, and speech therapy are important components of stroke rehabilitation and can help improve the patient’s mobility, speech, and overall quality of life. 

Therefore, the rehabilitation measures of massage, physical therapy, and speech therapy after a hemorrhagic stroke belong to tertiary prevention, which is option D. 


34. A 39-year-old woman went to the doctor for the first time with complaints of heavy menstruation for 10-12 days. The last menstruation started 3 weeks ago and continues until now. From the anamnesis: the cycle was broken 3 months ago, childbirth – 2, abortions – 5. On examination: the cervix is cylindrical, the eye is closed. Uterus and appendages without pathological changes. Discharges are bloody, excessive. What tactics of the doctor will be most appropriate? Select one:

A. Hemostatic therapy

B. Supravaginal amputation of the uterus

C. Cryocoagulation of the endometrium

D. Hormonal therapy

E. Scraping of the uterine cavity


Answer: Scraping the uterine cavity

Explanation

The clinical presentation of the 39-year-old woman with heavy menstruation for 10-12 days, a prolonged menstrual cycle, and a history of broken cycles, childbirth, and abortions is suggestive of a possible endometrial pathology, such as endometrial hyperplasia or endometrial cancer. 

Given the patient’s history and examination findings, the most appropriate tactic for the doctor would be to perform a diagnostic curettage or scraping of the uterine cavity, which is option E.  This procedure involves removing a sample of the endometrial tissue for examination to determine the underlying cause of the abnormal bleeding. 

Hemostatic therapy, hormonal therapy, and cryocoagulation of the endometrium may provide temporary relief of symptoms, but they are unlikely to address the underlying cause of the abnormal bleeding. 

Supravaginal amputation of the uterus may be considered if the patient has confirmed endometrial cancer, but it is a radical procedure that should be reserved for cases where more conservative treatments have failed. Therefore, the most appropriate tactic for the doctor in this case would be to perform a diagnostic curettage or scraping of the uterine cavity, which is option E. 


35. A 24-year-old woman in labor is in the postpartum period, the 4th day. The birth took place first, on time, without complications. The general condition of the woman in labor is satisfactory. Body temperature – 36.6 C. Ps – 78/min., rhythmic. Mammary glands in a state of swelling. The bottom of the uterus is 2 cm below the navel. The uterus is normal, painless. Blood lochia, moderate. Defecation, urination is normal. What treatment should be prescribed? Select one:

A. Analgesics

B. Antibiotics

C. Estrogenic hormones

D. Uterotonic drugs

E. Drugs that stop lactation o


Answer: Uterotonic drugs

Explanation

In the postpartum period, the most common complication is postpartum hemorrhage, which can occur due to poor uterine contraction and retained placental tissue. Therefore, it is important to ensure that the uterus is well-contracted to prevent bleeding and promote healing. In this case, the 24-year-old woman is in the postpartum period on the 4th day, and her general condition is satisfactory. However, the uterus is normal and painless, which may indicate poor uterine contraction.  

Therefore, the most appropriate treatment in this case would be to prescribe uterotonic drugs, which are medications that stimulate uterine contractions and help prevent postpartum hemorrhage. This is option D. Analgesics may be prescribed for pain relief, but they are unlikely to improve uterine contraction. Antibiotics may be prescribed if there is an infection, but there is no indication of infection in this case.  

Estrogenic hormones are not recommended in the postpartum period and may increase the risk of thromboembolic events. Drugs that stop lactation are not necessary at this time, as lactation is a natural process that occurs after childbirth. Therefore, the most appropriate treatment for the 24-year-old woman in this case would be to prescribe uterotonic drugs to promote uterine contraction and prevent postpartum hemorrhage, which is option D. 


36. A 49-year-old patient with a prolonged bronchial asthma attack was hospitalized. Weak breathing in the lungs, a lot of dry wheezing. ChDR – 32/min., BP – 140/90 mm Hg, Ps – 90/min. An intravenous injection of 1 ml of adrenaline was made. The condition has not improved: shortness of breath remains at 28/min., the number of dry wheezes in the lungs has decreased. Blood pressure rose to 170/110 mm Hg, tachycardia 130/min. To provide emergency aid, it is advisable to use: Select one:

A. Intravenous verapamil

B. β-blockers IV

C. Corinfarus rozzhuvati

D. Atrovent through a nebulizer

E. Cordaron internally


Answer : Verapamil IV

Explanation

In this case, the 49-year-old patient with a prolonged bronchial asthma attack has not responded to an intravenous injection of adrenaline, and their blood pressure has increased to 170/110 mm Hg with tachycardia. Therefore, it is important to provide emergency treatment to address the patient’s symptoms and prevent further complications. 

The most appropriate emergency treatment in this case would be to administer intravenous verapamil, which is a calcium channel blocker that can help lower blood pressure and heart rate. This is option A. β-blockers are contraindicated in patients with asthma, as they can exacerbate airway obstruction and bronchospasm.

Corinfar is a calcium channel blocker, similar to verapamil, but it is not available in all countries. Atrovent through a nebulizer may help improve airway obstruction, but it is unlikely to address the patient’s high blood pressure and tachycardia. Cordarone is an antiarrhythmic medication and is not indicated for the treatment of asthma attacks. 

Therefore, the most appropriate emergency treatment for this patient would be to administer intravenous verapamil to lower blood pressure and heart rate, which is option A. 


37. A 55-year-old patient complains of defecation 3-4 times within 1-2 hours, mainly in the morning, after breakfast. Increased diarrhea is associated with increased demands from the environment, anxiety, and anticipation of danger. Objectively: fussy, multilingual, overweight. The skin is unchanged, the turgor is normal, the abdomen is soft and painless. There are no changes in the blood test; stool analysis: unformed, a small amount of starch grains and muscle fibers, leukocytes – 3-4 in p/z; colonoscopy without pathology. Which drug would be most appropriate to prescribe? Select one:

A. Anaprilin

B. Ampicillin

C. Loperamide

D. Bisacodyl

E. Drotaverin


Answer : Loperamide

Explanation

The 55-year-old patient is presenting with symptoms of diarrhea, which is associated with anxiety and anticipation of danger. The patient is also overweight and fussy, but the physical exam and stool analysis do not reveal any significant abnormalities.

Therefore, the most likely diagnosis is functional diarrhea, which is diarrhea that occurs without any underlying organic cause. In this case, the most appropriate drug to prescribe would be loperamide, which is an antidiarrheal medication that works by slowing down bowel movements and reducing the frequency and amount of stool.

This is option C. Anaprilin is a beta-blocker that is not indicated for the treatment of diarrhea. Ampicillin is an antibiotic and is not appropriate for the treatment of functional diarrhea. Bisacodyl is a laxative and is not indicated for the treatment of diarrhea. Drotaverin is a smooth muscle relaxant and is not indicated for the treatment of diarrhea. 

Therefore, the most appropriate drug to prescribe for this patient would be loperamide, which is an antidiarrheal medication that can help reduce the frequency and amount of stool in functional diarrhea. 


38. A 68-year-old patient turned to the doctor with complaints about the presence of a tumor in the left mammary gland. During the examination in the upper inner quadrant of the left mammary gland, there is a mass up to 2.5 cm in diameter, dense, lumpy, painless during palpation. Regional lymph nodes are not enlarged. What is the most likely diagnosis? Select one:

A. Mastopathy

B. Cyst

C. Cancer

D. Fibroadenoma

E. Lipoma


Answer: Раĸ

Explanation

The presence of a mass in the left mammary gland of a 68-year-old patient that is dense, lumpy, and painless during palpation suggests a high likelihood of malignancy. In addition, the mass is up to 2.5 cm in diameter, which is larger than what is typically seen in benign breast conditions such as cysts or fibroadenomas. 

Therefore, the most likely diagnosis in this case would be breast cancer, which is option C. It is important to note that breast cancer can present in different ways and that the definitive diagnosis can only be made with a biopsy and histological examination of the tissue. 

Mastopathy, cysts, fibroadenomas, and lipomas are all benign breast conditions that can present with a mass in the breast, but they are typically smaller in size and may have different characteristics on palpation and imaging. However, it is important to rule out these conditions and perform further diagnostic tests to confirm the diagnosis of breast cancer. 

Therefore, in this case, further testing, such as a biopsy or mammogram, would be necessary to confirm the diagnosis of breast cancer and determine the appropriate treatment plan.  It is important to note that the final decision should be made by a healthcare professional based on a comprehensive evaluation of the individual case and the clinical findings at the time of labor.


39. A 72-year-old patient turned to the doctor with complaints of tremors in the hands, more in the right, general stiffness, slowness of gait, speech, difficulty in performing usual housework. During the physical examination, general bradykinesia, hypomimia, emprostotonus, “shuffling” gait, acheirokinesis, rest tremor in the hands, D>S, increased muscle tone of the plastic type, “cogwheel” phenomenon, pronounced postural instability. What drugs, first of all, should be in the treatment scheme of this patient? Select one:

A. Cholinolytics

B. Anticholinesterase drugs

C. Medicines that improve microcirculation

D. Medicines containing levodopa

E. Antidepressants


Answer : Medicines containing levodopa

Explanation

The 72-year-old patient is presenting with symptoms that are suggestive of Parkinson’s disease, including tremors in the hands, stiffness, slowness of gait and speech, difficulty performing housework, bradykinesia, hypomimia, and rest tremor. The physical exam findings of emprostotonus, “shuffling” gait, acheirokinesis, increased muscle tone of the plastic type, “cogwheel” phenomenon, and postural instability further support this diagnosis. 

The first-line treatment for Parkinson’s disease is medications containing levodopa, which is a precursor to dopamine that can help improve the symptoms of Parkinson’s disease. This is option D. Cholinolytics and anticholinesterase drugs are not indicated for the treatment of Parkinson’s disease and may worsen the symptoms.

Medicines that improve microcirculation and antidepressants are also not indicated for the treatment of Parkinson’s disease. Therefore, the most appropriate drugs to include in the treatment scheme for this patient would be medications containing levodopa, which can help improve the symptoms of Parkinson’s disease and improve the patient’s quality of life.


40. The patient developed an itchy skin rash 4 days ago. Abdominal bloating, dull pain in the right hypochondrium, constipation bothers. The day before, she ate smoked meat. Previously, similar reactions appeared after eating a significant amount of tomatoes, strawberries, chocolate. Objectively: urticarial rash on the skin of the face, trunk, limbs. The level of total IgE is normal. What is the most likely diagnosis? Select one:

A. Chronic urticaria

B. Pseudoallergy

C. Idiosyncrasy

D. Food allergy

E. Atopic dermatitis


Answer: Pseudoallergy

Explanation

The patient developed an itchy skin rash, abdominal bloating, dull pain in the right hypochondrium, and constipation after eating smoked meat.  The patient also reports a history of similar reactions after eating tomatoes, strawberries, and chocolate. The physical exam reveals an urticarial rash on the skin of the face, trunk, and limbs, and the level of total IgE is normal. 

Based on these findings, the most likely diagnosis would be pseudoallergy, which is a non-immunological reaction to a food or drug that can produce symptoms similar to those seen in an allergic reaction.  The symptoms can include hives, itching, abdominal pain, and diarrhea, and they can occur within minutes to hours after exposure to the trigger. 

Chronic urticaria is a type of skin rash characterized by the presence of hives that persist for more than 6 weeks. Idiosyncrasy refers to an individual’s unique reaction to a substance that is not related to an immunological response.  

Food allergy is an immunological reaction to a specific food allergen, which can produce symptoms such as hives, itching, abdominal pain, and diarrhea. Atopic dermatitis is a chronic skin condition characterized by dry, itchy, and inflamed skin. 

Therefore, based on the history and physical exam findings, the most likely diagnosis, in this case, would be pseudoallergy, which is a non-immunological reaction to a food trigger that can produce symptoms similar to those seen in an allergic reaction.


41. The parents of a 3-month-old child turned to the orthopedist with complaints about the asymmetry of the skin folds of the right thigh. The examination revealed: a different number of skin folds on the right and left thigh, their asymmetry, limitation of extension of the right lower limb bent at an angle of 90 in the hip and knee joints. What is the most likely diagnosis? Select one:

A. Dysplasia of the hip joint

B. –

C. Damage to the hip joint during childbirth

D. Congenital hip dislocation

E. Contracture of the hip joint o


Answer : Dysplasia of the hip joint

Explanation

The parents of a 3-month-old child have brought the child to an orthopedist with complaints about the asymmetry of the skin folds of the right thigh. The examination reveals a different number of skin folds on the right and left thigh, their asymmetry, and limitation of extension of the right lower limb bent at an angle of 90 in the hip and knee joints.  

Based on these findings, the most likely diagnosis is dysplasia of the hip joint, which is a condition where the hip joint does not develop properly and can result in dislocation or subluxation of the hip joint. The asymmetry of skin folds and limitation of extension in the hip and knee joints are signs of hip dysplasia.

This is option A. Damage to the hip joint during childbirth can also cause hip dysplasia but is less likely in this case since the child is already 3 months old. Congenital hip dislocation is a more severe form of hip dysplasia where the head of the femur is completely displaced from the acetabulum, and contracture of the hip joint can be a consequence of untreated hip dysplasia. 

Therefore, the most likely diagnosis in this case is dysplasia of the hip joint, and further testing and treatment, such as an ultrasound or referral to a pediatric orthopedist, may be necessary to confirm the diagnosis and determine the appropriate management. Therefore, in this case, measuring the level of IgE would be the most relevant immunologic index to assess the allergic response in the patient.


42. A 43-year-old patient was hospitalized 2 months after tonsillitis with complaints of shortness of breath, heart pain, dizziness, palpitations. Objectively: the general condition is difficult. Breathing rate – 35/min., pulse – 100/min., arrhythmic (extrasystole). Blood pressure – 145/60 mm Hg. Heart sounds are arrhythmic (extrasystole), weakening of the first sound, systolic murmur over the apex. The liver is enlarged. ECG: sinus tachycardia, single ventricular extrasystoles. Echocardiography of the heart cavity is within normal limits, FV-50%. What is the most likely diagnosis? Select one:

A. Dilated cardiomyopathy

B. Dressler syndrome

C. Infectious pericarditis

D. Infectious myocarditis

E. Myocardial cardiofibrosis


Answer: Infectious myocarditis

Explanation

The 43-year-old patient was hospitalized 2 months after tonsillitis with complaints of shortness of breath, heart pain, dizziness, palpitations, and an overall difficult condition. The physical exam reveals tachypnea, arrhythmic pulse (extrasystole), elevated blood pressure, weakened first heart sound, systolic murmur over the apex, and an enlarged liver.

The ECG shows sinus tachycardia with single ventricular extrasystoles, and echocardiography of the heart cavity is within normal limits with a ventricular ejection fraction of 50%. Based on these findings, the most likely diagnosis is infectious myocarditis, which is an inflammatory disease of the heart muscle caused by a viral, bacterial, fungal or parasitic infection.

The symptoms of shortness of breath, heart pain, arrhythmic pulse, systolic murmur, and elevated liver enzymes are typical of myocarditis. The history of recent tonsillitis suggests a possible infectious cause. Dilated cardiomyopathy is a condition characterized by a weakened and enlarged heart muscle, which can lead to heart failure. Dressler syndrome is a type of pericarditis that occurs after a heart attack.

Infectious pericarditis is inflammation of the pericardium, the sac surrounding the heart, and can present with similar symptoms but without the systolic murmur. Myocardial cardiofibrosis is a rare condition characterized by fibrosis of the heart muscle. 

Therefore, based on the history, physical exam, and diagnostic tests, the most likely diagnosis in this case would be infectious myocarditis, and treatment, such as antiviral or immunosuppressive therapy, may be necessary to manage the symptoms and prevent further complications. not typically present with acute symptoms of infection. D. Anthrax is caused by the bacterium Bacillus anthracis and usually presents with specific skin lesions called eschars, which are not mentioned in the scenario. 

E. Tularemia, caused by the bacterium Francisella tularensis, can cause swollen lymph nodes and fever, but the other symptoms described in the scenario are not typical of tularemia. It is important to consult a healthcare professional for an accurate diagnosis and appropriate treatment.


43. In an eight-month-old child, tonic tension of facial muscles, carpopedal spasm, and laryngospasm are noted, followed by generalized clonic convulsions with loss of consciousness for several minutes. Positive symptoms of Khvostek, Trousseau, Lust. What emergency condition is observed in the child?

a. Epileptic attack

b. Hypocalcemic (tetanic) convulsions

c. Affective respiratory convulsions

d. Choking and cyanotic attack

e. Febrile convulsions


Answer : Hypocalcemic (tetanic) convulsions

Explanation

In an eight-month-old child presenting with tonic tension of facial muscles, carpopedal spasm, laryngospasm, generalized clonic convulsions, and positive symptoms of Khvostek, Trousseau, and Lust, the most likely diagnosis is hypocalcemic (tetanic) convulsions. Hypocalcemic convulsions occur due to low levels of calcium in the blood, which can cause increased neuromuscular excitability and lead to muscle spasms and convulsions.

The positive symptoms of Khvostek, Trousseau, and Lust are indicative of hypocalcemia, which is a deficiency of calcium in the blood. These symptoms are caused by the stimulation of the facial nerve, the contraction of the muscles of the hand and wrist, and the spasm of the laryngeal muscles, respectively. Epileptic attacks typically present with different symptoms and do not have positive symptoms of Khvostek, Trousseau, or Lust.

Affective respiratory convulsions are a rare condition that occurs in infants and is characterized by episodes of apnea and cyanosis during crying or emotional stress. Choking and cyanotic attacks can occur due to airway obstruction. Febrile convulsions occur in response to a high fever and typically occur in children between 6 months and 5 years of age. 

Therefore, based on the symptoms and positive signs of hypocalcemia, the most likely diagnosis in this case is hypocalcemic (tetanic) convulsions, which is a medical emergency that requires prompt treatment to prevent further complications. In an eight-month-old child presenting with tonic tension of facial muscles, carpopedal spasm, laryngospasm, generalized clonic convulsions, and positive symptoms of Khvostek, Trousseau, and Lust, the most likely diagnosis is hypocalcemic (tetanic) convulsions. 

Hypocalcemic convulsions occur due to low levels of calcium in the blood, which can cause increased neuromuscular excitability and lead to muscle spasms and convulsions. The positive symptoms of Khvostek, Trousseau, and Lust are indicative of hypocalcemia, which is a deficiency of calcium in the blood. These symptoms are caused by the stimulation of the facial nerve, the contraction of the muscles of the hand and wrist, and the spasm of the laryngeal muscles, respectively. 

Epileptic attacks typically present with different symptoms and do not have positive symptoms of Khvostek, Trousseau, or Lust. Affective respiratory convulsions are a rare condition that occurs in infants and is characterized by episodes of apnea and cyanosis during crying or emotional stress. Choking and cyanotic attacks can occur due to airway obstruction. Febrile convulsions occur in response to a high fever and typically occur in children between 6 months and 5 years of age. 

Therefore, based on the symptoms and positive signs of hypocalcemia, the most likely diagnosis in this case is hypocalcemic (tetanic) convulsions, which is a medical emergency that requires prompt treatment to prevent further complications. In an eight-month-old child presenting with tonic tension of facial muscles, carpopedal spasm, laryngospasm, generalized clonic convulsions, and positive symptoms of Khvostek, Trousseau, and Lust, the most likely diagnosis is hypocalcemic (tetanic) convulsions. 

Hypocalcemic convulsions occur due to low levels of calcium in the blood, which can cause increased neuromuscular excitability and lead to muscle spasms and convulsions. The positive symptoms of Khvostek, Trousseau, and Lust are indicative of hypocalcemia, which is a deficiency of calcium in the blood. These symptoms are caused by the stimulation of the facial nerve, the contraction of the muscles of the hand and wrist, and the spasm of the laryngeal muscles, respectively. 

Epileptic attacks typically present with different symptoms and do not have positive symptoms of Khvostek, Trousseau, or Lust. Affective respiratory convulsions are a rare condition that occurs in infants and is characterized by episodes of apnea and cyanosis during crying or emotional stress.

Choking and cyanotic attacks can occur due to airway obstruction. Febrile convulsions occur in response to a high fever and typically occur in children between 6 months and 5 years of age. Therefore, based on the symptoms and positive signs of hypocalcemia, the most likely diagnosis in this case is hypocalcemic (tetanic) convulsions, which is a medical emergency that requires prompt treatment to prevent further complications. 


44. A 45-year-old man turned to the doctor with complaints of irritability, increased fatigue, weight loss, palpitations, heart failure. When palpating the left lobe of the thyroid gland, a dense, elastic mass is palpable, painless, which moves along with the thyroid gland when swallowing. Positive eye symptoms, exophthalmos, convergence disorder. Blood pressure – 135/80 mm Hg, pulse – 110/min. An ultrasound examination of the thyroid gland in the left lobe visualizes a hyperechoic rounded mass measuring 2 x 3 cm, with clear contours and a homogeneous structure. What conclusion of the doctor after the examination will be the most correct? Select one:

A. Thyroid nodule

B. Acute thyroiditis

C. Thyroid cancer

D. Thyroid cyst

E. Diffuse toxic goiter


Answer : Thyroid nodule

Explanation

The 45-year-old man presents with irritability, increased fatigue, weight loss, palpitations, heart failure, and a palpable, painless, elastic mass in the left lobe of the thyroid gland that moves along with the thyroid gland when swallowing.  Positive eye symptoms, exophthalmos, and convergence disorder are also noted, along with a blood pressure of 135/80 mm Hg and a pulse of 110/min.

An ultrasound examination of the thyroid gland reveals a hyperechoic rounded mass measuring 2 x 3 cm with clear contours and a homogeneous structure. Based on these findings, the most likely conclusion of the doctor after the examination would be a thyroid nodule, which is a solid or fluid-filled lump that forms within the thyroid gland.  

The symptoms of irritability, increased fatigue, weight loss, palpitations, and heart failure, along with the presence of eye symptoms and exophthalmos, suggest that the thyroid nodule may be associated with hyperthyroidism. 

Acute thyroiditis is an inflammation of the thyroid gland and is not consistent with the presentation in this case. Thyroid cancer is a possibility but would need to be confirmed through further testing such as a biopsy.  

A thyroid cyst is a fluid-filled sac within the thyroid gland, and diffuse toxic goiter is an enlargement of the thyroid gland associated with hyperthyroidism. Therefore, based on the symptoms and diagnostic findings, the most likely conclusion in this case would be a thyroid nodule, and further testing and management, such as a biopsy or thyroid function tests, may be necessary to determine the appropriate treatment. can only be made by a healthcare professional based on a thorough evaluation of the patient’s symptoms, medical history, and diagnostic tests.



45. A 46-year-old woman, who has been suffering from hypertension for 5 years, developed a hypertensive crisis. Complaints about palpitations, a feeling of pulsation in the head, heart rate – 100/min., blood pressure – 190/100 mm Hg. (hyperkinetic type of hemodynamics). Which drug should be preferred? Select one:

A. β-blocker

B. α-blocker

C. Dihydropyridine calcium antagonist

D. Diuretic

E. ACE inhibitor


Answer: β-blocker

Explanation

In a 46-year-old woman with a history of hypertension for 5 years who has developed a hypertensive crisis with complaints of palpitations, a feeling of pulsation in the head, heart rate of 100/min, and blood pressure of 190/100 mm Hg (hyperkinetic type of hemodynamics), the preferred drug would be a β-blocker. β-blockers decrease heart rate and reduce blood pressure by blocking the effects of the hormone epinephrine, which causes the heart to beat faster and harder, and constricts blood vessels, leading to increased blood pressure.

In hypertensive crisis, a β-blocker is preferred because it can quickly reduce heart rate and blood pressure, and prevent heart damage, stroke, and other complications. α-blockers, dihydropyridine calcium antagonists, diuretics, and ACE inhibitors are also used in the treatment of hypertension, but they may not be as effective in rapidly reducing blood pressure in a hypertensive crisis. α-blockers can cause reflex tachycardia and may not be the best option for a patient with palpitations and a high heart rate.  

Dihydropyridine calcium antagonists may not be as effective in reducing heart rate and may cause peripheral edema. Diuretics may take longer to reduce blood pressure and may cause electrolyte imbalances.

ACE inhibitors are not recommended in acute hypertensive crisis due to the risk of hypotension. Therefore, based on the presentation of the patient, a β-blocker would be the preferred drug to manage the hypertensive crisis, and further monitoring and management, such as blood pressure checks and adjustment of medication dosages, may be necessary to manage the patient’s hypertension in the long term. C. Acute glomerulonephritis is an inflammation of the glomeruli (filtering units) in the kidneys, which typically presents with symptoms such as hematuria (blood in the urine) and proteinuria (elevated protein levels in the urine).

However, the symptoms described by the girl, including suprapubic pain and frequent urination, are more indicative of bladder involvement rather than glomerular inflammation. D. Acute pyelonephritis involves an infection and inflammation of the kidneys. While it can cause symptoms similar to cystitis, such as painful urination, the absence of significant flank pain and a negative Pasternatsky symptom make acute cystitis a more likely diagnosis. E. Urolithiasis refers to the presence of urinary stones in the urinary tract.

While it can cause symptoms such as suprapubic pain and blood in the urine, the presence of a urinary tract infection with associated symptoms suggests acute cystitis as the primary diagnosis. It’s important to note that a definitive diagnosis can only be made by a healthcare professional based on a comprehensive evaluation of the patient’s symptoms, medical history, physical examination, and additional diagnostic tests if necessary.


46. The city is provided with drinking water from underground artesian springs, which is characterized by high quality and purity. Which of the listed methods of treatment of water obtained from underground interlayer pressure sources is provided by sanitary requirements? Select one:

A. Coagulation

B. Decontamination

C. Bleaching

D. Desalination

E. Defluorination


Answer : Decontamination

Explanation

The method of treatment of water obtained from underground interlayer pressure sources that is provided by sanitary requirements is decontamination. Decontamination is the process of removing or reducing harmful substances, such as bacteria, viruses, parasites, and other microorganisms, from water to make it safe for human consumption. This is an important step in ensuring that drinking water is free from harmful contaminants that could cause disease or illness. 

Coagulation is a process that involves adding chemicals to water to form particles, which then settle out of the water. This process can help remove suspended solids and some organic material, but it may not be effective in removing all harmful contaminants. Bleaching is a process of adding chemicals such as chlorine or other oxidizing agents to water to kill microorganisms and disinfect the water.

This process may be effective in reducing some harmful contaminants but may not remove all of them. Desalination is a process of removing salt and other minerals from water to make it safe for human consumption. This process is typically used for seawater or brackish water and may not be necessary for water obtained from underground interlayer pressure sources. Defluorination is a process of removing excess fluoride from water, which may be necessary in areas where the natural fluoride content of the water exceeds safe levels.

However, this may not be necessary for all sources of drinking water. Therefore, based on the information provided, the method of treatment of water obtained from underground interlayer pressure sources that is provided by sanitary requirements is decontamination, which is necessary to ensure that the water is safe for human consumption by removing or reducing harmful contaminants. 


47. A 28-year-old patient discovered a tumor in the upper outer quadrant of the right mammary gland during a self-examination. During palpation, there is a painless, hard, mobile mass in the mammary gland with a diameter of 2 cm, the peripheral lymph nodes are not changed. Ultrasound examination of the mammary glands: in the upper outer quadrant of the right mammary gland, a volumetric formation of increased echogenicity, 21 x 18 mm in size. What is the most likely diagnosis? Select one:

A. Breast cyst

B. Diffuse mastopathy

C. Mastitis

D. Fibroadenoma

E. Breast cancer


Answer : Fibroadenoma

Explanation

In a 28-year-old patient who discovered a tumor during a self-examination in the upper outer quadrant of the right mammary gland, the most likely diagnosis based on the presentation and diagnostic findings is a fibroadenoma. Fibroadenomas are the most common benign breast tumors and are typically painless, hard, mobile masses in the breast with a smooth, well-defined border. They are most commonly found in women in their 20s and 30s and are characterized by an increase in glandular and connective tissue.  

Fibroadenomas can range in size from a few millimeters to several centimeters and are typically discovered during breast self-examination or routine clinical breast examination. In this case, the patient has a painless, hard, mobile mass in the mammary gland with a diameter of 2 cm, and an ultrasound examination of the mammary glands reveals a volumetric formation of increased echogenicity, 21 x 18 mm in size, which is consistent with a fibroadenoma.

The absence of changes in the peripheral lymph nodes also suggests a benign lesion. Breast cysts are fluid-filled sacs that may be palpable and may cause pain or tenderness, but they typically do not present as a hard mass. Diffuse mastopathy is a benign condition characterized by changes in breast tissue, but it does not typically present as a discrete mass.  

Mastitis is an infection of the breast tissue that can cause pain, swelling, and redness, but it typically presents with other symptoms such as fever and flu-like symptoms. Breast cancer is a possibility, but a biopsy would be necessary to confirm the diagnosis. Therefore, based on the presentation and diagnostic findings, the most likely diagnosis in this case is a fibroadenoma, a common benign breast tumor that typically does not require surgical intervention but may need to be monitored over time.


48. The city has a centralized drinking water supply system. Its source is a surface reservoir – a river, which according to water quality indicators belongs to the II class of water sources. During the current laboratory control of water quality at the point “before the water enters the external distribution network”, the local SES recorded two consecutive deviations of water quality according to the indicators of epidemic safety. What is the most likely reason for the deterioration of water quality according to epidemic indicators? Select one:

A. Unsatisfactory operation of water treatment (main) facilities

B. Unsatisfactory transportation of the sample to the laboratory

C. Deterioration of the sanitary condition of the reservoir – river

D. Violation of sampling rules

E. Stagnation of water in the distribution network


Answer: Unsatisfactory operation of water treatment (main) facilities

Explanation

In a 28-year-old patient who discovered a tumor during a self-examination in the upper outer quadrant of the right mammary gland, the most likely diagnosis based on the presentation and diagnostic findings is a fibroadenoma. Fibroadenomas are the most common benign breast tumors and are typically painless, hard, mobile masses in the breast with a smooth, well-defined border. They are most commonly found in women in their 20s and 30s and are characterized by an increase in glandular and connective tissue.

Fibroadenomas can range in size from a few millimeters to several centimeters and are typically discovered during breast self-examination or routine clinical breast examination. In this case, the patient has a painless, hard, mobile mass in the mammary gland with a diameter of 2 cm, and an ultrasound examination of the mammary glands reveals a volumetric formation of increased echogenicity, 21 x 18 mm in size, which is consistent with a fibroadenoma.

The absence of changes in the peripheral lymph nodes also suggests a benign lesion. Breast cysts are fluid-filled sacs that may be palpable and may cause pain or tenderness, but they typically do not present as a hard mass. Diffuse mastopathy is a benign condition characterized by changes in breast tissue, but it does not typically present as a discrete mass. Mastitis is an infection of the breast tissue that can cause pain, swelling, and redness, but it typically presents with other symptoms such as fever and flu-like symptoms.

Breast cancer is a possibility, but a biopsy would be necessary to confirm the diagnosis. Therefore, based on the presentation and diagnostic findings, the most likely diagnosis in this case is a fibroadenoma, a common benign breast tumor that typically does not require surgical intervention but may need to be monitored over time.  In a 28-year-old patient who discovered a tumor during a self-examination in the upper outer quadrant of the right mammary gland, the most likely diagnosis based on the presentation and diagnostic findings is a fibroadenoma. 

Fibroadenomas are the most common benign breast tumors and are typically painless, hard, mobile masses in the breast with a smooth, well-defined border.  They are most commonly found in women in their 20s and 30s and are characterized by an increase in glandular and connective tissue. Fibroadenomas can range in size from a few millimeters to several centimeters and are typically discovered during breast self-examination or routine clinical breast examination. 

In this case, the patient has a painless, hard, mobile mass in the mammary gland with a diameter of 2 cm, and an ultrasound examination of the mammary glands reveals a volumetric formation of increased echogenicity, 21 x 18 mm in size, which is consistent with a fibroadenoma. The absence of changes in the peripheral lymph nodes also suggests a benign lesion. Breast cysts are fluid-filled sacs that may be palpable and may cause pain or tenderness, but they typically do not present as a hard mass.

Diffuse mastopathy is a benign condition characterized by changes in breast tissue, but it does not typically present as a discrete mass.  Mastitis is an infection of the breast tissue that can cause pain, swelling, and redness, but it typically presents with other symptoms such as fever and flu-like symptoms. Breast cancer is a possibility, but a biopsy would be necessary to confirm the diagnosis. Therefore, based on the presentation and diagnostic findings, the most likely diagnosis in this case is a fibroadenoma, a common benign breast tumor that typically does not require surgical intervention but may need to be monitored over time. 


49. A 42-year-old patient complains of a painful mass in the left buttock, an increase in body temperature to 38.2°C. A week ago, she completed a course of treatment for lumbar sciatica. Hyperemia, swelling in the upper outer quadrant of the left buttock. On palpation – sharply painful compaction up to 6 cm in diameter with softening in the center. Your previous diagnosis: Select one:

A. Purulent atheroma

B. Buttock carbuncle

C. Post-injection abscess

D. Abscessive furuncle

E. Subcutaneous paraproctitis


Answer : Post-injection abscess

Explanation

Based on the presentation of the patient, the most likely diagnosis is a post-injection abscess. Post-injection abscesses can occur as a complication of injections, especially if the injection site is not properly disinfected or if the needle is contaminated. The symptoms can include pain, swelling, redness, and fever. The abscess may be located deep under the skin and may feel like a hard, painful mass.In this case, the patient complains of a painful mass in the left buttock, with hyperemia and swelling in the upper outer quadrant of the left buttock.

On palpation, there is a sharply painful compaction up to 6 cm in diameter with softening in the center. These findings are typical of an abscess. The recent completion of a course of treatment for lumbar sciatica may have involved injections, which may have contributed to the development of the abscess.Purulent atheroma, buttock carbuncle, abscessive furuncle, and subcutaneous paraproctitis may also present with similar symptoms, but they are less likely in this case.

Purulent atheroma is a benign tumor of the sebaceous gland, buttock carbuncle is a collection of boils under the skin, abscessive furuncle is a localized infection of a hair follicle, and subcutaneous paraproctitis is an infection of the tissue around the rectum. None of these conditions are specifically related to injections.

Therefore, based on the presentation of the patient and the symptoms described, the most likely diagnosis is a post-injection abscess, which may require treatment with antibiotics and/or drainage of the abscess.


50. A 35-year-old woman turned to the doctor with complaints of increasing pain during menstruation during the 1st year. The duration of the menstrual cycle is 28 days. Laparoscopic tubal ligation was performed 2 years ago. On palpation, the uterus is enlarged to 8 weeks of pregnancy, soft, painful. The appendages on both sides are not enlarged, they are painless on palpation. What is the most likely diagnosis? Select one:

A. Hydrosalpinx

B. Adenomyosis

C. Ectopic pregnancy

D. Polycystic ovaries

E. External endometriosis


Answer : Adenomyosis

Explanation

Based on the presentation of the patient, the most likely diagnosis is adenomyosis. Adenomyosis is a condition in which the tissue that normally lines the inside of the uterus grows into the muscular wall of the uterus. This can cause the uterus to become enlarged, soft, and painful, especially during menstruation.

Women with adenomyosis may also experience heavy or prolonged menstrual bleeding, and the condition can lead to infertility. In this case, the patient complains of increasing pain during menstruation, and on palpation, the uterus is enlarged to 8 weeks of pregnancy, soft, and painful. These symptoms are consistent with adenomyosis.

The fact that the patient had a laparoscopic tubal ligation 2 years ago suggests that she has completed her childbearing and may be a candidate for hysterectomy as a treatment option. Hydrosalpinx is a condition in which the fallopian tube becomes blocked and filled with fluid, typically as a result of an infection or inflammation. Ectopic pregnancy is a pregnancy that occurs outside of the uterus, typically in the fallopian tube.

Polycystic ovaries is a condition in which the ovaries contain multiple small cysts, which can cause irregular periods, infertility, and other symptoms. External endometriosis is a condition in which endometrial tissue grows outside of the uterus, typically in the pelvis, and can cause pain, bleeding, and infertility.

None of these conditions are specifically related to the symptoms described by the patient. Therefore, based on the presentation of the patient and the symptoms described, the most likely diagnosis is adenomyosis, a condition in which the tissue that lines the inside of the uterus grows into the muscular wall of the uterus, causing the uterus to become enlarged, soft, and painful, especially during menstruation. 

Join the conversation
0% Complete